1985 AJHSME Problems/Problem 2

Revision as of 21:25, 12 January 2009 by 5849206328x (talk | contribs) (New page: ==Problem== <math>90+91+92+93+94+95+96+97+98+99=</math> <math>\text{(A)}\ 845 \qquad \text{(B)}\ 945 \qquad \text{(C)}\ 1005 \qquad \text{(D)}\ 1025 \qquad \text{(E)}\ 1045</math> ==So...)
(diff) ← Older revision | Latest revision (diff) | Newer revision → (diff)

Problem

$90+91+92+93+94+95+96+97+98+99=$


$\text{(A)}\ 845 \qquad \text{(B)}\ 945 \qquad \text{(C)}\ 1005 \qquad \text{(D)}\ 1025 \qquad \text{(E)}\ 1045$

Solution

This problem needs a solution. If you have a solution for it, please help us out by adding it.

See Also

1985 AJHSME Problems